Difference between revisions of "Hermite (physicist)"

From specialfunctionswiki
Jump to: navigation, search
(Properties)
Line 20: Line 20:
 
<strong>Theorem:</strong> (Orthogonality) Let $w(x)=e^{-x^2}$, then
 
<strong>Theorem:</strong> (Orthogonality) Let $w(x)=e^{-x^2}$, then
 
$$\displaystyle\int_{-\infty}^{\infty} H_n(x)H_m(x)w(x) dx=\sqrt{\pi}2^nn!\delta_{mn},$$
 
$$\displaystyle\int_{-\infty}^{\infty} H_n(x)H_m(x)w(x) dx=\sqrt{\pi}2^nn!\delta_{mn},$$
where $H_n$ denotes the [[Hermite (physicist)|Hermite polynomials]] and $\delta_{mn}$ denotes the [[Dirac delta]].
+
where $H_n$ denotes the [[Hermite (physicist)|Hermite polynomials]] and $\delta_{mn}$ denotes the [[Kronecker delta]].
 
<div class="mw-collapsible-content">
 
<div class="mw-collapsible-content">
 
<strong>Proof:</strong> █  
 
<strong>Proof:</strong> █  

Revision as of 22:39, 8 July 2016

The (physicist) Hermite polynomials are a sequence of orthogonal polynomials with weight function $e^{-x^2}$.

$$\begin{array}{ll} H_0(x)=1\\ H_1(x)=2x\\ H_2(x)=4x^2-2\\ H_3(x)=8x^3-12x\\ H_4(x)=16x^4-48x^2+12\\ H_5(x)=32x^5-160x^3+120x \end{array}$$

Properties

Theorem: (Orthogonality) Let $w(x)=e^{-x^2}$, then $$\displaystyle\int_{-\infty}^{\infty} H_n(x)H_m(x)w(x) dx=\sqrt{\pi}2^nn!\delta_{mn},$$ where $H_n$ denotes the Hermite polynomials and $\delta_{mn}$ denotes the Kronecker delta.

Proof:

Theorem: The following formula holds: $$H_{n+1}(x)=2xH_n(x)-H_n'(x).$$

Proof:

Theorem: The following formula holds: $$H_n'(x)=2nH_{n-1}(x).$$

Proof:

Theorem: The following formula holds: $$H_{n+1}(x)=2xH_n(x)-2nH_{n-1}(x).$$

Proof:


Orthogonal polynomials